$$\newcommand{\mtn}{\mathbb{N}}\newcommand{\mtns}{\mathbb{N}^*}\newcommand{\mtz}{\mathbb{Z}}\newcommand{\mtr}{\mathbb{R}}\newcommand{\mtk}{\mathbb{K}}\newcommand{\mtq}{\mathbb{Q}}\newcommand{\mtc}{\mathbb{C}}\newcommand{\mch}{\mathcal{H}}\newcommand{\mcp}{\mathcal{P}}\newcommand{\mcb}{\mathcal{B}}\newcommand{\mcl}{\mathcal{L}} \newcommand{\mcm}{\mathcal{M}}\newcommand{\mcc}{\mathcal{C}} \newcommand{\mcmn}{\mathcal{M}}\newcommand{\mcmnr}{\mathcal{M}_n(\mtr)} \newcommand{\mcmnk}{\mathcal{M}_n(\mtk)}\newcommand{\mcsn}{\mathcal{S}_n} \newcommand{\mcs}{\mathcal{S}}\newcommand{\mcd}{\mathcal{D}} \newcommand{\mcsns}{\mathcal{S}_n^{++}}\newcommand{\glnk}{GL_n(\mtk)} \newcommand{\mnr}{\mathcal{M}_n(\mtr)}\DeclareMathOperator{\ch}{ch} \DeclareMathOperator{\sh}{sh}\DeclareMathOperator{\th}{th} \DeclareMathOperator{\vect}{vect}\DeclareMathOperator{\card}{card} \DeclareMathOperator{\comat}{comat}\DeclareMathOperator{\imv}{Im} \DeclareMathOperator{\rang}{rg}\DeclareMathOperator{\Fr}{Fr} \DeclareMathOperator{\diam}{diam}\DeclareMathOperator{\supp}{supp} \newcommand{\veps}{\varepsilon}\newcommand{\mcu}{\mathcal{U}} \newcommand{\mcun}{\mcu_n}\newcommand{\dis}{\displaystyle} \newcommand{\croouv}{[\![}\newcommand{\crofer}{]\!]} \newcommand{\rab}{\mathcal{R}(a,b)}\newcommand{\pss}[2]{\langle #1,#2\rangle} $$
Bibm@th

Capes : exercices sur les suites

Pour réviser
Enoncé
Étudier la nature des suites suivantes, et déterminer leur limite éventuelle : $$\begin{array}{lcl} \displaystyle \mathbf 1.\ u_n=\frac{\sin(n)+3\cos\left(n^2\right)}{\sqrt{n}}&&\displaystyle \mathbf 2.\ u_n=\frac{2n+(-1)^n}{5n+(-1)^{n+1}}\\[0.1cm] \displaystyle \mathbf 3.\ u_n=\frac{n^3+5n}{4n^2+\sin(n)+\ln(n)}&&\displaystyle \mathbf 4.\ u_n= \sqrt{2n+1}-\sqrt{2n-1}\\[0.1cm] \displaystyle \mathbf 5.\ u_n=3^ne^{-3n}.&&\displaystyle \mathbf 6.\ u_n=\frac{n^3+2^n}{n^2+3^n}. \end{array}$$
Indication
Corrigé
Exercice 2 - Fonction décroissante - avec indications [Signaler une erreur] [Ajouter à ma feuille d'exos]
Enoncé
Soit $f:]0,+\infty[\to]0,+\infty[$ définie par $f(x)=1+\frac{2}{x}$. On considère la suite récurrente $(u_n)$ vérifiant $u_{n+1}=f(u_n)$ et $u_0=1$.
  1. Étudier le sens de variation de $f$ sur $[1,3]$ et montrer que l'intervalle $[1,3]$ est stable par $f$. Que peut-on en déduire sur $(u_n)$?
  2. Soit $(v_n)$ et $(w_n)$ les suites définies par $v_{n}=u_{2n}$ et $w_n=u_{2n+1}$. Montrer que $(v_n)$ est croissante.
  3. Démontrer que $(w_n)$ est décroissante.
  4. En déduire que $(v_n)$ et $(w_n)$ sont convergentes et déterminer leur limite respective.
  5. Quelle est la nature de la suite $(u_n)$?
Indication
Corrigé
Enoncé
Soient $(u_n)$ et $(v_n)$ deux suites réelles. Dire si les assertions suivantes sont vraies ou fausses. Lorsqu'elles sont vraies, les démontrer. Lorsqu'elles sont fausses, donner un contre-exemple.
  1. Si $(u_n)$ et $(v_n)$ divergent, alors $(u_n+v_n)$ diverge.
  2. Si $(u_n)$ et $(v_n)$ divergent, alors $(u_n\times v_n)$ diverge.
  3. Si $(u_n)$ converge et $(v_n)$ diverge, alors $(u_n+v_n)$ diverge.
  4. Si $(u_n)$ converge et $(v_n)$ diverge, alors $(u_n\times v_n)$ diverge.
  5. Si $(u_n)$ n'est pas majorée, alors $(u_n)$ tend vers $+\infty$.
  6. Si $(u_n)$ est positive et tend vers 0, alors $(u_n)$ est décroissante à partir d'un certain rang.
Indication
Corrigé
Exercice 4 - Avec des quantificateurs [Signaler une erreur] [Ajouter à ma feuille d'exos]
Enoncé
Soit $(u_n)$ une suite de nombres réels. Écrire avec des quantificateurs les propositions suivantes :
  1. $(u_n)$ est bornée.
  2. $(u_n)$ n'est pas croissante.
  3. $(u_n)$ n'est pas monotone.
  4. $(u_n)$ n'est pas majorée.
  5. $(u_n)$ ne tend pas vers $+\infty$.
Indication
Corrigé
Enoncé
Voici un extrait d'une copie d'élève :
Proposition : Soit $(u_n)$ une suite d'éléments de l'intervalle $]0,1[$. Alors la suite $(u_n^n)$ converge vers 0.
Démonstration : Pour tout $n \geq 1$, on a $u_n <1$. L'intervalle $]u_n, 1[$ est donc non vide, donc il existe $a$ tel que $0<u_n < a < 1$. Sur $\mathbb{R}^+$, la fonction $x \mapsto x^n$ est croissante, donc on a $0<u_n^n<a^n$. Comme $0<a<1$, on sait que $\displaystyle\lim_{n \to +\infty} a^n = 0$ (en effet, $a^n = e^{n \ln(a)}$ et vu que $0<a<1$, on a $\ln(a)<0$ donc $n \ln(a) \to -\infty$ quand $n \to +\infty$).
Par le théorème des gendarmes, vu que $0<u_n^n<a^n$ et que $\displaystyle\lim_{n \to +\infty} a^n = 0$, on en déduit que $\displaystyle\lim_{n \to +\infty} u_n^n = 0$.
  1. Démontrer que la proposition énoncée est fausse.
  2. Expliquer de façon claire et précise l'erreur commise.
Corrigé
Pour progresser
Exercice 6 - Une suite définie comme étant la racine d'un polynôme [Signaler une erreur] [Ajouter à ma feuille d'exos]
Enoncé
Pour $n\geq 1$, on considère le polynôme $P_n(X)=X^n+X^{n-1}+\dots+X-1$.
  1. Démontrer que $P_n$ possède une seule racine dans $\mathbb R_+$, que l'on note $u_n$.
  2. Démontrer que la suite $(u_n)$ est décroissante, et en déduire qu'elle converge.
  3. Démontrer que, pour tout $n\geq 1$, $u_n\geq\frac 12$.
  4. Soit $\rho\in ]1/2,1[$. Démontrer que $\lim_{n\to+\infty}P_n(\rho)>0$.
  5. Démontrer que $(u_n)$ converge vers $\frac 12$.
Indication
Corrigé
Enoncé
Soient $(u_n)$ et $(v_n)$ les deux suites définies pour $n\geq 1$ par : $$u_n=\sum_{k=0}^n\frac{1}{k!},\ \ v_n=u_n+\frac{1}{n\times n!}.$$
  1. Montrer que les suites $(u_n)$ et $(v_n)$ sont adjacentes. On note $e$ leur limite commune.
  2. Montrer que, pour tout $n\in\mathbb N^*$, $n! u_n< n! e < n!u_n+\frac 1n.$
  3. En déduire que $e$ est un nombre irrationnel (on pourra procéder par l'absurde).
  4. Écrire une fonction $\verb+approx(ecart)+$ sous Python qui renvoie un encadrement de $e$ avec une amplitude inférieure à ecart.
Indication
Corrigé
Enoncé
On définit deux suites $(p_n)$ et $(q_n)$ par les formules suivantes : $$\left\{ \begin{array}{rcl} p_0&=&\displaystyle \frac{3\sqrt 3}2\\ q_0&=&3\sqrt 3\\ q_{n+1}&=&\displaystyle \frac{2p_nq_n}{p_n+q_n}\\ p_{n+1}&=&\displaystyle \sqrt{p_n q_{n+1}} \end{array} \right. $$ On admettra que les suites $(p_n)$ et $(q_n)$ sont bien définies et vérifient, pour tout $n\in\mathbb N$, $p_n>0$ et $q_n>0$. Pour les quatre premières questions, on fera très attention à la nécessité, ou non, d'utiliser un raisonnement par récurrence.
  1. Montrer que, pour tout $n\in\mathbb N$, on a $p_n\leq q_n$.
  2. En déduire que la suite $(q_n)$ est décroissante.
  3. Montrer que, pour tout $n\in\mathbb N$, on a $p_n\leq q_{n+1}$.
  4. En déduire que la suite $(p_n)$ est croissante.
    1. Vérifier que, si $x,y$ sont des réels strictement positifs, alors $\frac{2xy}{x+y}\leq\frac 12(x+y)$.
    2. En déduire que, pour tout entier naturel $n$, on a $$q_{n+1}-p_{n+1}\leq \frac 12\left(q_n-p_n\right).$$
  5. Démontrer que les suites $(p_n)$ et $(q_n)$ sont adjacentes. On note $\ell$ leur limite.
  6. Écrire un algorithme donnant un encadrement de $\ell$ à $10^{-10}$ près.
  7. Dans la suite, on souhaite déterminer la valeur de $\ell$ (et donner une explication géométrique à la construction de ces deux suites). On se place dans le plan muni d'un repère orthonormé. Soit $\theta\in [0,\pi]$ et $A(\theta)$ le point d'affixe $e^{i\theta}$. Démontrer que la distance $A(0)A(\theta)$ vaut $2\sin(\theta/2)$.
  8. Pour $n\in\mathbb N$, on note $u_n$ la moitié du périmètre d'un polygone régulier inscrit dans le cercle unité à $3\times 2^n$ côtés. Démontrer que $$u_n=3\times 2^n\times\sin(a_n)$$ où $a_n=\frac{\pi}{3\times 2^n}$.
  9. On définit de même la suite $(v_n)$ pour $n\in\mathbb N$ par $$v_n=3\times 2^n\times\tan(a_n).$$ On démontre que, pour $n\in\mathbb N$, $v_n$ est la moitié du périmètre d'un polygone régulier à $3\times 2^n$ côtés dont le cercle inscrit est le cercle unité.
    Vérifier que, pour tout $n\in\mathbb N$, $$u_{n+1}=\sqrt{u_n v_{n+1}}\textrm{ et }v_{n+1}=\frac{2u_nv_n}{u_n+v_n}.$$
  10. Que peut-on en déduire sur les suites $(u_n)$, $(v_n)$, $(p_n)$ et $(q_n)$?
  11. Quelle est la limite commune des suites $(p_n)$ et $(q_n)$?
Indication
Corrigé
Enoncé
Soit $(u_n)_{n\geq 1}$ une suite réelle. On pose $S_n=\frac{u_1+\dots+u_n}{n}$.
  1. On suppose que $(u_n)$ converge vers 0. Soient $\veps>0$ et $n_0\in\mathbb N^*$ tel que, pour $n\geq n_0$, on a $|u_n|\leq\veps$.
    1. Montrer qu'il existe une constante $M$ telle que, pour $n\geq n_0$, on a $$|S_n|\leq \frac{M(n_0-1)}{n}+\veps.$$
    2. En déduire que $(S_n)$ converge vers 0.
  2. On suppose que $u_n=(-1)^n$. Que dire de $(S_n)$? Qu'en déduisez-vous?
  3. On suppose que $(u_n)$ converge vers $l$. Montrer que $(S_n)$ converge vers $l$.
  4. On suppose que $(u_n)$ tend vers $+\infty$. Montrer que $(S_n)$ tend vers $+\infty$.
Indication
Corrigé
Exercice 10 - Toute suite croissante et majorée converge [Signaler une erreur] [Ajouter à ma feuille d'exos]
Enoncé
Le but de cet exercice est de démontrer que toute suite croissante et majorée converge.
  1. Rappeler la définition de la borne supérieure d'une partie $A$ de $\mathbb R$.
  2. La condition "$A$ est non-vide et majorée" est-elle une condition nécessaire pour que $A\subset \mathbb R$ admette une borne supérieure? une condition suffisante? une condition nécessaire et suffisante?
  3. Les ensembles suivants admettent-ils une borne supérieure? Si oui, la déterminer. $$A=\left\{\frac 1n;\ n\in\mathbb N^*\right\},\ B=\left\{1-\frac 1n;\ n\in\mathbb N^*\right\}.$$
  4. Soit $(u_n)_{n\in\mathbb N}$ une suite croissante de réels, majorée. On note $A=\{u_n;\ n\in\mathbb N\}$ et $\ell =\sup A$. Soit $\veps>0$. Justifier qu'il existe $n_0\in\mathbb N$ tel que $u_{n_0}\in [\ell-\veps;\ell]$.
  5. En déduire que $(u_n)_{n\in\mathbb N}$ converge vers $\ell$.
Indication
Corrigé